Đến nội dung

viet9a14124869 nội dung

Có 855 mục bởi viet9a14124869 (Tìm giới hạn từ 29-04-2020)



Sắp theo                Sắp xếp  

#695856 $\sum \frac{a}{a^3+b^2+c}\leq 1$

Đã gửi bởi viet9a14124869 on 30-10-2017 - 21:55 trong Bất đẳng thức và cực trị

Cho a,b,c >0; a+b+c =3 .Chứng minh: $\sum \frac{a}{a^3+b^2+c}\leq 1$

Bài này cách hay dùng nhất chắc là dùng Bất Đẳng Thức Bunyakovsky :

$(a^3+b^2+c)(\frac{1}{a}+1+c)\geq (a+b+c)^2=9\rightarrow \frac{a}{a^3+b^2+c}\leq \frac{1+a+ac}{9}$

Làm tương tự rồi cộng các đẳng thức lại với nhau , kết hợp với đẳng thức $(a+b+c)^2\geq 3(ab+bc+ca)$ ( AM-GM )

$\sum \frac{a}{a^3+b^2+c}\leq \frac{3+a+b+c+ab+bc+ca}{9}\leq \frac{3+a+b+c+\frac{(a+b+c)^2}{3}}{9}=1$

Vậy ta có đpcm , dấu bằng xảy ra $\Leftrightarrow a=b=c=1$




#695852 Tìm cách giải mới

Đã gửi bởi viet9a14124869 on 30-10-2017 - 21:30 trong Phương trình - hệ phương trình - bất phương trình

Hi, mình có bài toán sau:

Tìm m để x đạt giá trị lớn nhất với m>0,x>0,n>0 thỏa phương trình $\sqrt{m}=mx^{n}+x$.

Các bạn có thể giải bài này k?

Mình thử chém tạm :v

Do m,x,n >0 nên $\sqrt{m}=mx^n+x\Leftrightarrow m=(mx^n+x)^2\Leftrightarrow x^{2n}.m^2+(2x^{n+1}-1)m+x^2=0$ (1)

Xét phương trình (1) ẩn m tham số x có $\Delta =1-4x^{n+1}\geq 0\rightarrow x_{max}=\sqrt[n+1]{\frac{1}{4}}$ ( cái này mình cũng không chắc lắm :v )

Từ đó tính được m theo n .




#695616 Cho a,b,c là các số thực thỏa mãn $1/a^2+1/b^2+1/c^2\geq 3/4$

Đã gửi bởi viet9a14124869 on 26-10-2017 - 21:54 trong Bất đẳng thức và cực trị

Cho a,b,c là các số thực thỏa mãn $1/a^2+1/b^2+1/c^2\geq 3/4$ . Chứng minh rằng 

$(a+b)/(ab+c)+(b+c)/(bc+a)+(a+c)/(ac+b)\geq 12/a+b+c$

Sửa đề đi bác :v hình như bác Châu đăng bài này trên face nhưng điều kiện phải là dấu $\leq $ chứ :v

Xin phép chỉ nêu ý tưởng để mọi người tham khảo :

Nhân thêm a+b ,b+c ,c+a vào tử và mẫu 3 phân số , áp dụng BĐT Schwarz rồi đổi biến p=a+b+c , q=ab+bc+ca , r=abc

Sau đó dùng BĐT Schur $p^3+9r \geq 4pq$ để đưa về chứng minh $ p \geq 6 $ ( Cái này dùng AM-GM cho điều kiện là ra ngay ý mà  :3 )  ^_^ 




#695472 $x+y+z+\frac{3(n-3)xyz}{xy+yz+zx}\geq n\sqrt{\frac{x...

Đã gửi bởi viet9a14124869 on 25-10-2017 - 19:47 trong Bất đẳng thức - Cực trị

Lời giải.

Do đẳng thức trên là thuần nhất nên ta có thể chuẩn hóa $q=3.$ Đổi biến $p=a+b+c,q=ab+bc+ca,r=abc.$

Ta đi chứng minh $p+(n-3)r \geq n.$

_Nếu $p^2 > 4q = 12 \rightarrow p > 2\sqrt{3}\rightarrow p+(n-3)r > p > 2\sqrt{3}> \frac{16}{5}\geq n$ ( đpcm )

_Nếu $p^2\leq 4q =12 \Rightarrow p \in [3;2\sqrt{3}].$

Áp dụng bất đẳng thức Schur ta có : $p^3+9r\geq 4pq=12p\Rightarrow r\geq \frac{12p-p^3}{9}\rightarrow p+(n-3)r-n\geq p+(n-3)(\frac{12p-p^3}{9})-n=(p-3)\left [1-\frac{(n-3)(p^2+3p-3)}{9} \right ]\geq 0 \Leftrightarrow 1-\frac{(n-3)(p^2+3p-3)}{9}\geq 0$

Thật vậy , vì $n\in \left \{ 3;\frac{16}{5} \right \}$ nên ta có : 

 $1-\frac{(n-3)(p^2+3p-3)}{9}\geq 1-\frac{\frac{1}{5}(p^2+3p-3)}{9}=\frac{48-3p-p^2}{45}\geq 0$  ( $\forall p \in \left [ 3 ; 2\sqrt{3} \right ]$ )

Vậy bài toán được chứng minh.

P/S : Đoán là bài này có thể làm mạnh hơn bằng cách cho $n\in \left [ 3; \frac{18}{5} \right ]$ nhưng chưa kiểm chứng  :biggrin:




#695467 Chứng minh bất đẳng thức $\dfrac{a}{b}+\df...

Đã gửi bởi viet9a14124869 on 25-10-2017 - 19:20 trong Bất đẳng thức và cực trị

Cho $a,b,c$ là các số thực dương thỏa mãn $\sqrt{a}+\sqrt{b}+\sqrt{c}=3$. Chứng minh rằng

$$\dfrac{a}{b}+\dfrac{b}{c}+\dfrac{c}{a}\ge ab+bc+ac$$

Áp dụng bất đẳng thức Cauchy cho bộ 3 số ta có : $\left\{\begin{matrix} a^2c+ab^2+ab^2 \geq 3ab\sqrt[3]{abc} & & \\ ab^2+bc^2+bc^2 \geq 3bc\sqrt[3]{abc} & & \\ bc^2+a^2c+a^2c \geq 3ca\sqrt[3]{abc} & & \end{matrix}\right.$

Cộng vế theo vế các đẳng thức trên ta suy ra được : $\frac{a}{b}+\frac{b}{c}+\frac{c}{a}=\frac{a^2c+bc^2+b^2a}{abc}\geq \frac{(ab+bc+ca)\sqrt[3]{abc}}{abc}\geq ab+bc+ca \Leftrightarrow \sqrt[3]{abc}\geq abc$ hay $1\geq abc $

Điều này đúng vì $3=\sqrt{a}+\sqrt{b}+\sqrt{c}\geq 3\sqrt[6]{abc}\Rightarrow 1\geq abc$ ( thỏa mãn )

Vậy bài toán được chứng minh xong , dấu bằng xảy ra khi a=b=c=1 !




#695299 $$\frac{1}{x}+\frac{1}...

Đã gửi bởi viet9a14124869 on 23-10-2017 - 20:16 trong Bất đẳng thức - Cực trị

Cho $x,y,z>0$ thoả mãn $x+y+z+xyz=4$,chứng minh rằng:

$$\frac{1}{x}+\frac{1}{y}+\frac{1}{z}\geqslant \left ( \frac{17\sqrt{17}-47}{8} \right )(x+y+z)+\frac{165-51\sqrt{17}}{8}$$

Mình thấy bài này hình như không chỉ có 1 dấu bằng tại 3 biến bằng nhau :  x=y=z=1

Cho thử x=y=t , khi đó ta có $z=\frac{4-2t}{t^2+1}$ ( t > 0 )

Vậy ta sẽ tìm t để $\frac{2}{t}+\frac{t^2+1}{4-2t}=\frac{17\sqrt{17}-47}{8}(2t+\frac{4-2t}{t^2+1})+\frac{165-51\sqrt{17}}{8}$

Phương trình này chuyển về thành phương trình bậc 5 , ngoài nghiệm t=1 còn ra 1 nghiệm nữa là t=0,831987.....

Liệu dấu bằng này có đúng không ạ ??? Mọi người cho mình xin ít ý kiến với ???  ^_^ 




#695090 $\frac{a+b}{\sqrt{ab+c}}+\frac{b+c}{\sqrt{bc+a}}+...

Đã gửi bởi viet9a14124869 on 19-10-2017 - 20:23 trong Bất đẳng thức và cực trị

cho a, b, c > 0 và $\frac{1}{a}+\frac{1}{b}+\frac{1}{c}\geqslant 1$

Chứng minh rằng $\frac{a+b}{\sqrt{ab+c}}+\frac{b+c}{\sqrt{bc+a}}+\frac{c+a}{\sqrt{ca+b}}\geq 3\sqrt[6]{abc}$

Từ điều kiện bài toán ta có : $abc\leq ab+bc+ca$

Ta có $\frac{a+b}{\sqrt{ab+c}}\geq \frac{(a+b)\sqrt{c}}{\sqrt{(a+c)(b+c)}}$

Làm tương tự ta được :

$LHS \geq \frac{(a+b)\sqrt{c}}{\sqrt{(c+a)(c+b)}}+\frac{(b+c)\sqrt{a}}{\sqrt{(a+b)(a+c)}}+\frac{(c+a)\sqrt{b}}{\sqrt{(b+c)(b+a)}}\geq 3\sqrt[6]{abc}$ ( AM-GM )

 

P/S: Bài này dễ mà :v ( Đã fix )




#693329 $a^2+b^2+4ab+16\ge4c^2-16c+20$ Thi vào 10 LHP Nam Định (Đề Chung)

Đã gửi bởi viet9a14124869 on 18-09-2017 - 22:19 trong Bất đẳng thức và cực trị

Cho $a,b,c$ là các số thực thỏa mãn $a\ge -2$, $b\ge -2$ và $a+b+2c=6$. Chứng minh rằng

a) $a^2+b^2+4ab+16\ge4c^2-16c+20$.

b) $\dfrac{4-b^2}{4\left[(c-2)^2+1\right]}-\dfrac{a^2}{(a-b)^2+6ab+16}+5\ge 0$.

 

Đề thi vào 10 Chuyên Lê Hồng Phong Nam Định 2017 - 2018 (Đề chung)

Hmm....

a, Do điều kiện bài toán nên ta có a+b=6-2c và $(a+2)(b+2)\geq 0 \rightarrow a^2+b^2+4ab+16=(a+b)^2+2(a+2)(b+2)-4(a+b)+8\geq (a+b)^2-4(a+b)+8=(6-2c)^2-4(6-2c)+8=4c^2-16c+20$ ( Q.E.D)

Đẳng thức xảy ra khi a=-2 hoặc b=-2

b, Theo a, ta thấy $(a-b)^2+6ab+16\geq 4c^2-16c+20$

Do đó $A=\frac{4-b^2}{4c^2-16c+20}-\frac{a^2}{(a-b)^2+6ab+16}+5\geq \frac{4-b^2-a^2}{4c^2-16c+20}+5=\frac{4-(a+b)^2+2(a+2)(b+2)-4(a+b)-8}{4c^2-16c+20}+5\geq \frac{4-(a+b)^2-4(a+b)-8}{4c^2-16c+20}+5=\frac{4-(6-2c)^2-4(6-2c)-8}{4c^2-16c+20}+5=\frac{4(2c-3)^2}{4c^2-16c+20}\geq 0$ ( Q.E.D)

Dấu bằng xảy ra khi $(a,b,c)\in \left \{ (-2,5,\frac{3}{2});(5,-2,\frac{3}{2}) \right \}$




#693151 Đề chọn Đội tuyển HSGQG tỉnh Hòa Bình năm 2017-2018

Đã gửi bởi viet9a14124869 on 16-09-2017 - 18:40 trong Thi HSG cấp Tỉnh, Thành phố. Olympic 30-4. Đề thi và kiểm tra đội tuyển các cấp.


 

Câu 2 : Cho đường tròn tâm $O$ đường kính $AB$ . Một điểm $H$ thuộc đoạn $AB$ . Đường thẳng qua $H$ vuông góc với $AB$ cắt $(O)$ tại $C$ . Đường tròn đường kính $CH$ cắt $AC,CB,(O)$ tại $D,E,F$

 

a) Chứng minh rằng $AB,DE,CF$ đồng quy

 

b) Đường tròn tâm $C$ bán kính $CH$ cắt $(O)$ tại $P,Q$

Chứng minh rằng $P,Q,D,E$ thẳng hàng

 

Trình của em làm đề này chỉ được 5 điểm :v

a, Giả sử CF cắt BA tại X , khi đó ta có CFDE , CFAB và DABE là tứ giác nội tiếp

Ta thấy $\widehat{XFD}=\widehat{CED}=\widehat{DAB}\rightarrow$ FXAD là tứ giác nội tiếp

Do đó $\widehat{XDA}=\widehat{XFA}=\widehat{CBX}=\widehat{CDE}\rightarrow \overline{X,D,E}$ ( đpcm )

b, Lấy P,Q là giao điểm của DE với ( O) ao cho D nằm giữa P và E

Theo phần a ta có $\overline{X,P,D,E,Q}$

Do CFPA VÀ XFDA là tứ giác nội tiếp nên $\widehat{CPF}=\widehat{CAF}=\widehat{CXD}$

Suy ra $\Delta CPF\sim \Delta CXP$ ( g-g ) $\rightarrow CP^2=CF.CX$ (1)

Theo hệ thức lượng trong tam giác vuông , $CH^2=CF.CX$ (2)

Từ (1) và (2) suy ra CP=CH . Chứng minh tương tự ta có CQ=CH

Vậy ta có đpcm .




#692952 ĐỀ THI LẬP ĐỘI TUYỂN TOÁN LỚP 12 DAKLAK

Đã gửi bởi viet9a14124869 on 12-09-2017 - 21:36 trong Thi HSG cấp Tỉnh, Thành phố. Olympic 30-4. Đề thi và kiểm tra đội tuyển các cấp.

Bài 1 : $\left\{\begin{matrix} \frac{x}{\sqrt{y^2+1}}+\frac{y}{\sqrt{x^2+1}}=\frac{x+y}{\sqrt{1+xy}} & & \\ \sqrt{(2x-2)(y+5)}+\sqrt{(2y-2)(x+2)}-3(\sqrt{y+5}+\sqrt{x+2})=3 & & \end{matrix}\right.$

ĐKXĐ: xy $\geq$ -1

Nếu y=-5 thì ta suy ra $\left\{\begin{matrix} x+2\geq 0 & & \\ x+2\leq 0& & \end{matrix}\right.\rightarrow x=-2$ .Thế vào hệ đầu thì suy ra vô lí .

Nếu x=-2 thì ta cũng suy ra được điều tương tự .

Vậy ta có ĐKXĐ :  x,y >1

Xét phương trình thứ nhất , ta chứng minh LHS $\geq$ RHS .

Thật vậy $LHS =\frac{x}{\sqrt{y^2+1}}+\frac{y}{\sqrt{x^2+1}}\geq \frac{(x+y)^2}{x\sqrt{y^2+1}+y\sqrt{x^2+1}}\geq \frac{(x+y)^2}{\sqrt{2(2x^2y^2+x^2+y^2)}}\geq \frac{x+y}{\sqrt{1+xy}}\Leftrightarrow (xy-1)(x-y)^2\geq 0$ ( đúng vì x > 1 và y > 1 )

Do đó LHS =RHS $\Leftrightarrow $ x=y

Thay x=y vào phương trình thứ 2 , suy ra $(\sqrt{2x-2}-3)(\sqrt{x+5}+\sqrt{x+2})=3\Leftrightarrow \sqrt{2x-3}-3=\sqrt{x+5}-\sqrt{x+2}$

$\Leftrightarrow \frac{x-7}{\sqrt{2x-2}+\sqrt{x+5}}=\frac{7-x}{3+\sqrt{x+2}}\Leftrightarrow x=7$

Vậy x=y=7 là nghiệm của hệ .




#692858 Chứng minh rằng với mọi số thực dương a,b,c ta luôn có ...

Đã gửi bởi viet9a14124869 on 11-09-2017 - 18:58 trong Bất đẳng thức và cực trị

Bài toán :

Chứng minh rằng với mọi số thực dương a,b,c ta luôn có :

            $(\sum a)(\sum \frac{1}{\sqrt{a}})\geq 2\sqrt[4]{216\sum(a^2+bc)}-\sum \sqrt{a}$

                                                                                          ---- NĐV - ĐQC ----




#692778 Tìm k

Đã gửi bởi viet9a14124869 on 10-09-2017 - 10:52 trong Số học

Gọi A là tập tất cả các số tự nhiên lẻ không chia hết cho 5 và nhỏ hơn 30. Tìm số k nguyên dương nhỏ nhất sao cho mỗi tập con của A gồm k phần tử đều tồn tại hai số, mà số này chi hết cho số kia.

Làm sai thì thôi nha

Ta thấy $A=\left \{ 1;3;7;9;11;13;17;19;21;23;27;29 \right \}$ gồm 12 phần tử

Vì tập A có tất cả 8 số nguyên tố nên ta chứng minh k nhỏ nhất bằng 9

Thật vậy , chọn tập con B của A gồm 9 phần tử bất kì

Xét các cặp số (1,13) (3,9)(7,21) .Dễ thấy hai số trong cùng 1 cặp thì là bội của nhau ;)

Do vậy nếu tập B chứa 2 số trong 1 cặp bất kì thì ta có đpcm

Nếu B không cùng chứa 2 số trong bất kì cặp số nào ở trên thì do B có 9 phần tử nên B sẽ chứa 1 số trong mỗi cặp trên và 6 số còn lại ( 3,11,17,19,27,29 )

Suy ra trong B phải có cặp ( 3,27) hoặc (9,27 ) ( thỏa mãn yêu cầu bài toán )

Vậy phép chứng minh hoàn tất , k = 9 . %%-




#692725 $\left\{\begin{matrix} (x-1)y^{2...

Đã gửi bởi viet9a14124869 on 09-09-2017 - 21:09 trong Phương trình - hệ phương trình - bất phương trình

1) $\left\{\begin{matrix} (x-1)y^{2}+x+y=3 & \\ (y-2)x^{2}+y=x+1 & \end{matrix}\right.$

2) $\left\{\begin{matrix} y^{4}-xy^{3} +x^{2}y^{2}=16& \\ y^{2}-xy^{3}-xy =4 & \end{matrix}\right.$

3)$\left\{\begin{matrix} x+y-\sqrt{xy}=15 & \\ \sqrt{x+1}+\sqrt{y+1}=8 & \end{matrix}\right.$

1, Hệ đã cho tương đương $\left\{\begin{matrix} (x-1) (y^2+1)=2-y& & \\ (y-2)(x^2+1)=x-1 & & \end{matrix}\right.$

Nhân vế theo vế ta được $(x-1)(y-2)(x^2y^2+x^2+y^2+2)=0\Leftrightarrow$ x=1 hoặc y =2

+) Nếu x=1 thì y=2 và ngược lại .

Vậy ta có nghiệm của hệ là (1,2)

2, Hệ $\left\{\begin{matrix} y^4-xy^3+x^2y^2=16 & & \\ y^2-xy^3-xy=4 & & \end{matrix}\right.\Leftrightarrow \left\{\begin{matrix} (y^2-xy)^2+(y^2-xy)=20 & & \\ y^2-xy-xy^3=4 & & \end{matrix}\right.\Leftrightarrow \left\{\begin{matrix} y^2-xy=4 & & \\ y^2-xy-xy^3=4& & \end{matrix}\right.$ hoặc $\left\{\begin{matrix} y^2-xy=-5 & & \\ y^2-xy-xy^3=4 & & \end{matrix}\right.$

Xét 2 trường hợp là ra .




#692695 Cho 3 số thực dương a,b,c sao cho a+b+c=1 ..............

Đã gửi bởi viet9a14124869 on 09-09-2017 - 18:09 trong Bất đẳng thức và cực trị

cho 3 số thực dương a,b,c sao cho  $a+b+c=1; Tìm min  P=2(a^2b+b^2c+c^2a)+(a^2+b^2+c^2)+4abc$

Cách khác :

Không mất tính tổng quát ta giả sử a nằm giữa b và c .

Ta sẽ chứng minh $LHS=2(a^2b+b^2c+c^2a)+a^2+b^2+c^2+4abc\geq 2a^2(b+c)+a^2+b^2+c^2+2bc(a+b+c)\Leftrightarrow c(a-b)(c-a)\geq 0$ ( đúng )

Thay b+c=1-a ta đi tìm min của biểu thức $2a^2(b+c)+a^2+b^2+c^2+2bc(a+b+c)=2a^2(1-a)+a^2+b^2+c^2+2bc=2a^2(1-a)+a^2+(1-a)^2=\frac{19}{27}+\frac{2(4-3a)(3a-1)^2}{27}\geq \frac{19}{27}$ ( vì $1>a$ )

Vậy min = $\frac{19}{27}\Leftrightarrow a=b=c=\frac{1}{3}$ ^_^




#692687 Chứng minh rằng: $\frac{1}{3}n^2+\frac{1}{2}n+\frac{1}{6}...

Đã gửi bởi viet9a14124869 on 09-09-2017 - 16:31 trong Bất đẳng thức - Cực trị

Chứng minh rằng: $\frac{1}{3}n^2+\frac{1}{2}n+\frac{1}{6}\ge (n!)^{\frac{2}{n}}$ với $n\in \mathbb{N}^*$

Ta có một đẳng thức khá quen thuộc : $1^2+2^2+3^2+...+n^2=\frac{n(n+1)(2n+1)}{6}$

Ta chứng minh bằng cách THCS :

$1^2+2^2+3^2+...+n^2=1(2-1)+2(3-1)+3(4-1)+...+n(n+1-1)=[1.2+2.3+3.4+...+n(n+1)]-(1+2+3+...+n)=\frac{n(n+1)(n+2)-0.1.2}{3}-\frac{n(n+1)}{2}=\frac{n(n+1)(2n+1)}{6}$

Ngoài ra còn có thể chứng minh theo quy nạp hoặc dùng hằng đẳng thức các kiểu ....

Quay lại bài toán ,

+) Nếu n=1 ta có ngay đpcm .

+)Nếu n >1 ,áp dụng bất đẳng thức AM-GM ta có :

$RHS=\sqrt[n]{(n!)^2}=\sqrt[n]{1^2.2^2.3^2.....n^2}\leq \frac{1^2+2^2+3^2+...+n^2}{n}=\frac{n(n+1)(2n+1)}{6n}=\frac{(n+1)(2n+1)}{6}=\frac{1}{3}n^2+\frac{1}{2}n+\frac{1}{6}$

Chứng minh hoàn tất .




#692192 Sử dụng hàm số trong bài toán cực trị

Đã gửi bởi viet9a14124869 on 03-09-2017 - 10:08 trong Bất đẳng thức và cực trị

1, Cho các số thực dương x,y,z thỏa mãn x+y+z=4xyz. Chứng minh rằng:

$ \frac{1}{x(y+z)}+\frac{1}{y(z+x)}+\frac{1}{z(x+y)} > \frac{5}{x+y+z} $

2, Cho a,b,c là các số dương. CMR:

$ 3(a+b+c) \geq \sqrt[3]{\frac{a^{3}+b^{3}+c^{3}}{3}}+8\sqrt[3]{abc} $

3, Cho ba số thực không âm a,b,c thỏa mãn $ 5(a^{2}+b^{2}+c^{2})=6(ab+bc+ca) $

Tìm GTLN: $ P=\sqrt{2(a+b+c)}-(b^{2}+c^{2}) $

4, Cho x,y,z là các số dương thay đổi. Tìm GTNN của:

$ P=x(\frac{x}{2}+\frac{1}{yz})+y(\frac{y}{2}+\frac{1}{zx})+z(\frac{z}{2}+\frac{1}{xy}) $

5, Cho hai số thực x,y thay đổi thỏa mãn $ x^{2}+y^{2}=1 $. Tìm GTLN và GTNN của biểu thức: 

$ P=\frac{2(x^{2}+6xy)}{1+2xy+2y^{2}} $

Bài 1 mình chém 1 cách dùng p,q,r nhưng không hay lắm, chắc sẽ có cách khác :)

Bài 2 :

Áp dụng bất đẳng thức Holder ta có : $(\frac{\sqrt[3]{a^3+b^3+c^3}}{3}+8\sqrt[3]{abc})^3\leq (\frac{a^3+b^3+c^3}{3}+8abc).9^2=27(a^3+b^3+c^3+24abc)\leq 27(a+b+c)^3\Rightarrow Q.E.D$

Dấu bằng xảy ra khi a=b=c .

Bài 3 : Kì nhỉ liệu đề có phải là tìm max của $\sqrt{2(a+b+c)}-(a^2+b^2+c^2)$ ko ??! 

Bài 5 : Bài này dùng phương pháp miền giá trị , ta thấy được max=3 và min = - 6

Thật vậy ta có $3-P=3-\frac{2(x^2+6xy)}{x^2+2xy+3y^2}=\frac{(x-3y)^2}{x^2+2xy+3y^2}\geq 0$

                        $P+6=\frac{2(x^2+6xy)}{x^2+2xy+3y^2}+6=\frac{2(2x+3y)^2}{x^2+2xy+3y^2}\geq 0$

Dấu bằng 2 trường hợp thì bạn tự tìm nhé :)

P/S : CHƯA HỌC HÀM :v




#692174 MAX $\frac{3a+2b+c}{(a+b)(b+c)(c+a)}$

Đã gửi bởi viet9a14124869 on 03-09-2017 - 07:14 trong Bất đẳng thức - Cực trị

Tìm dấu bằng kiểu gì vậy anh ?

Giá trị lớn nhất là $\frac3{16}$ đạt được khi 3 biến đều bằng nhau.

 

$\left\{\begin{matrix} a,b,c>0 & \\ 3bc+4ca+5ab\leq 6abc & \end{matrix}\right.$

 

Tìm MAX:

 

$\frac{3a+2b+c}{(a+b)(b+c)(c+a)}$

Ta sẽ chứng minh một đẳng thức : $24abc(3a+2b+c)\leq (5ab+3bc+4ca)^2\Leftrightarrow 9(ab-bc)^2+16(ab-ca)^2\geq 0$ ( đúng )

Kêt hợp với giả thiết bài toán , $\Rightarrow 24abc(3a+2b+c)\leq (5ab+3bc+4ca)^2\leq 36a^2b^2c^2\Rightarrow 3a+2b+c\leq \frac{3}{2}abc$

Suy ra $P= \frac{3a+2b+c}{(a+b)(b+c)(c+a)}\leq \frac{3}{2}.\frac{abc}{(a+b)(b+c)(c+a)}\leq \frac{3}{16}$ ( đúng theo AM-GM )

Vậy max P =$\frac{3}{16}$ $\Leftrightarrow a=b=c=2$ :)




#691623 Kĩ thuật đổi biến trong chứng minh bất đẳng thức

Đã gửi bởi viet9a14124869 on 26-08-2017 - 21:24 trong Chuyên đề toán THCS

Topic thật sự rất hay , và rất tiếc là gần 1 năm rồi mà vẫn chưa có ai post lời giải ,mình sẽ trình bày vài bài mà mình có thể nghĩ ra :))

Bài 6: Cho $a,b,c$ là các số thực bất kỳ. Chứng minh rằng:

$\left ( \frac{a}{a-kb} \right )^2+\left ( \frac{b}{b-kc} \right )^2+\left ( \frac{c}{c-ka} \right )^2+\frac{2(1-k^3)abc}{(a-kb)(b-kc)(c-kb)}\geq 1$

trong đó $k$ là số thực sao cho $(a-kb)(b-kc)(c-kb)\neq 0$

Bài 7: Cho $a,b,c$ là các số thực đôi một khác nhau. Chứng minh rằng khi đó với mọi $k$, ta luôn có bất đẳng thức:

$\left ( \frac{a+kb}{a-b} \right )^2+\left ( \frac{b+kc}{b-c} \right )^2+\left ( \frac{c+ka}{c-a} \right )^2\geq k^2+1$

Spoiler

Bài toán 6 :

Đổi biến $x=\frac{a}{a-kb},y=\frac{b}{b-kc},z=\frac{c}{c-ka}$

Ta dễ có $x-1=\frac{kb}{a-kb}\Rightarrow (x-1)(y-1)(z-1)=\frac{k^3abc}{(a-kb)(b-kc)(c-ka)}$

Vậy đẳng thức cần chứng minh tương đương $x^2+y^2+z^2+2xyz-2(x-1)(y-1)(z-1)\geq1 \Leftrightarrow (x+y+z-1)^2\geq 0$ ( đúng )

Vậy ta có đpcm .

Bài toán 7 :

Xét trường hợp k = -1 , ta có ngay đpcm .

Nếu k khác -1 , đổi biến $x=\frac{a+kb}{a-b},y=\frac{b+kc}{b-c},z=\frac{c+ka}{c-a}$

Ta cần chứng minh : $x^2+y^2+z^2\geq k^2+1$

Mặt khác từ cách lấy x,y,z ta có :

$(x-1)(y-1)(z-1)=(x+k)(y+k)(z+k)\Leftrightarrow (k+1)(xy+yz+zx)+(k^2-1)(x+y+z)+k^3+1=0 \Leftrightarrow xy+yz+zx = -(k^2-k+1)+(1-k)(x+y+z)$

Vì $x^2+y^2+z^2-k^2-1=(x+y+z)^2-2(xy+yz+zx)-k^2-1=(x+y+z)^2+2(k-1)(x+y+z)+k^2-2k+1=(x+y+z+k-1)^2\geq 0$

Vậy ta có đpcm .

 

Bài 10 [Vasile Cirtoaje]

Cho a, b, c là những số thực dương thoả mãn: $(a+b+c)(\frac{1}{a}+\frac{1}{b}+\frac{1}{c})=13$

Tìm GTNN của biểu thức: $(a^2+b^2+c^2)(\frac{1}{a^2}+\frac{1}{b^2}+\frac{1}{c^2})$

Gợi ý: đặt: $x=\sum \frac{a}{b};y=\sum \frac{b}{a}$. Chú ý mối liên hệ giữa x và y để đưa BĐT cần chứng minh về BĐT mới chỉ gồm 2 biến x, y.

Giải bài 10 luôn :))

Đặt $x=\sum \frac{a}{b},y=\sum \frac{b}{a}\Rightarrow x+y+3=(a+b+c)(\frac{1}{a}+\frac{1}{b}+\frac{1}{c})=13\rightarrow x+y=10$

Ta có $A=(a^2+b^2+c^2)(\frac{1}{a^2}+\frac{1}{b^2}+\frac{1}{c^2})=3+\sum \frac{a^2}{b^2}+\sum\frac{b^2}{a^2}$

Mặt khác theo đánh giá $\sum \frac{a^2}{b^2}=x^2-2y,\sum \frac{b^2}{a^2}=y^2-2x$

Thì ta được $A=x^2+y^2-2x-2y+3\geq \frac{(x+y)^2}{2}-2.10+3=50-20+3=33$ ( AM-GM )

Vậy giá trị nhỏ nhất của A là 33 khi x=y=5 .




#691583 $\sum {\frac{{a - b}}{b}...

Đã gửi bởi viet9a14124869 on 26-08-2017 - 15:29 trong Bất đẳng thức - Cực trị

Cho 3 số dương a,b,c : CMR $\sum {\frac{{a - b}}{b}}  \ge \frac{{{{(a - c)}^2}}}{{(a + b)(b + c)}}$

Áp dụng bất đẳng thức Schwarz :

       $LHS =\frac{a}{b}+\frac{b}{c}+\frac{c}{a}-3=\frac{a^2}{ab}+\frac{b^2}{bc}+\frac{c^2}{ca}+\frac{b^2}{b^2}-4\geq \frac{(a+2b+c)^2}{b^2+ba+bc+ac}-4=\frac{(a-c)^2}{(a+b)(b+c)}$ = RHS .

Dấu "=" xảy ra khi a=b=c !




#691574 Cmr:$6+27(\sum ab)\leq 5\left [\sum \frac{...

Đã gửi bởi viet9a14124869 on 26-08-2017 - 13:08 trong Bất đẳng thức và cực trị

$\boxed{\text{Bài toán}}$:

Cho a,b,c là các số thực thỏa mãn $a+b+c=1$. Chứng minh rằng:

$6+27(ab+bc+ca)\leq 5\left [ \frac{a(3-4b^2-4c^2)}{a^2+1}+\frac{b(3-4c^2-4a^2)}{b^2+1}+\frac{c(3-4a^2-b^2)}{c^2+1} \right ]$

 

                                                                                                       $\boxed{\text{cristianoronaldo}}$

Bất đẳng thức sai tại $a=b=c=\frac{1}{3}$




#691371 CMR a, $\sqrt{(3-x)^{2}+3y^{2}}+...

Đã gửi bởi viet9a14124869 on 24-08-2017 - 08:06 trong Bất đẳng thức và cực trị

[quote name="BiBi Chi" post="691132" timestamp="1503218815"]CMR 
a, $\sqrt{(3-x)^{2}+3y^{2}}+\sqrt{(3+x)^{2}+3y^{2}}+\left | 9-2y \right |\geq 2\sqrt{6}+9$

Dùng phone nên hơi khó viết . Phương pháp làm câu a là dùng 2 bất đẳng thức Minkowski và Cauchy nhé.



#691330 Tìm GTNN:$P=\left ( 2-xyz \right )\left ( \sum\...

Đã gửi bởi viet9a14124869 on 23-08-2017 - 16:34 trong Bất đẳng thức và cực trị

$\boxed{\text{Bài toán}}$(cristianoronaldo)

Cho x,y,z là các số thực dương thỏa mãn $xy+yz+zx=3$.

Tìm GTNN của biểu thức:

$P=\left ( 2-xyz \right )\left ( \frac{1}{x\sqrt{y^2+z^2}}+\frac{1}{y\sqrt{z^2+x^2}}+\frac{1}{z\sqrt{x^2+y^2}} \right )$

Ta thấy $3=xy+yz+zx\geq 3\sqrt[3]{x^2y^2z^2}\Rightarrow 1\geq xyz$ và $x+y+z\geq \sqrt{3(xy+yz+zx)}=3$

Mặt khác áp dụng bất đẳng thức Cauchy ta có :

 $\sum x\sqrt{y^2+x^2}\leq \sqrt{3(2x^2y^2+2y^2z^2+2z^2x^2)}=\sqrt{6(9-2xyz(x+y+z))}\leq \sqrt{6(9-2xyz.3)}=3\sqrt{6-4xyz}$

Dùng bất đẳng thức Schwarz , suy ra 

$P=(2-xyz)(\sum \frac{1}{x\sqrt{y^2+z^2}})\geq (2-xyz).\frac{9}{\sum x\sqrt{y^2+z^2}}\geq \frac{9(2-xyz)}{3\sqrt{6-4xyz}}$

Ta sẽ chứng minh $P\geq \frac{3}{2}\Leftrightarrow \frac{9(2-xyz)}{3\sqrt{6-4xyz}}\geq \frac{3}{2}\Leftrightarrow (1-xyz)(2-xyz)\geq 0$ ( đúng ) 

Vậy ta có $min_{P}=\frac{3}{2}\Leftrightarrow x=y=z=1$ . 




#690821 $\sum \sqrt{1+\frac{48x}{y+z}}\geq 15$ .

Đã gửi bởi viet9a14124869 on 17-08-2017 - 22:16 trong Bất đẳng thức và cực trị

$\frac{1}{\sqrt{1+\frac{48x}{y+z}}+5}\geq \frac{1}{\sqrt{1+\frac{48y}{x+z}}+5}\geq \frac{1}{\sqrt{1+\frac{48z}{x+y}}+5}$

 

Chưa đủ trình dùng Chebyshev đâu Chính :) , dãy bôi đỏ ngược chiều rồi nhá :)




#690800 $\sum \sqrt{1+\frac{48x}{y+z}}\geq 15$ .

Đã gửi bởi viet9a14124869 on 17-08-2017 - 21:00 trong Bất đẳng thức và cực trị

Bài 2 chuẩn hóa $ x+y+z=3$ chẳng hạn sau đó sử dụng BĐT Jensen với $ f(x) = \sqrt{1+ \dfrac{48x}{3-x}}$

Có lời giải sơ cấp hơn không anh , em vẫn chưa học hàm :))




#690699 Cmr:$\frac{x+y}{z}+\frac{y+z}...

Đã gửi bởi viet9a14124869 on 16-08-2017 - 21:26 trong Bất đẳng thức và cực trị

$\boxed{\text{Bài toán}}$:

Cho x,y,z là các số thực dương thỏa mãn $xyz=1$. Chứng minh rằng:

$\frac{x+y}{z}+\frac{y+z}{x}+\frac{z+x}{y}\geq \sqrt{3\left ( x^3+y^3+z^3 \right )}+3$

 

                                                                                        $\boxed{\text{cristianoronaldo}}$

Đã ra hhehe :))

Không giảm tính tổng quát , giả sử x và y nằm cùng phía với 1 . Khi đó ta có :

$x^3+y^3\leq x^3y^3+1=\frac{1}{z^3}+1=\frac{z^3+1}{z^3}\Rightarrow VP \leq \sqrt{3(z^3+\frac{z^3+1}{z^3})}+3$

Mặt khác theo bất đẳng thức AM-GM ,ta có :

$VT =\frac{x+y}{z}+(\frac{z}{x}+\frac{z}{y})+(\frac{x}{y}+\frac{y}{x})\geq \frac{2\sqrt{xy}}{z}+2\sqrt{\frac{z^2}{xy}}+2=\frac{2}{z\sqrt{z}}+2z\sqrt{z}+2$

Vậy ta đi chứng minh $2z\sqrt{z}+\frac{2}{z\sqrt{z}}+2\geq \sqrt{3(z^3+\frac{z^3+1}{z^3})}+3\Leftrightarrow (z\sqrt{z}-1)^4\geq 0$ ( đúng )

Vậy ta có đpcm , dấu "=" xảy ra khi x=y=z=1 .